Download as pdf or txt
Download as pdf or txt
You are on page 1of 40

MANTHAN

PRELIMS TEST SERIES-2024

pPLT-3.0 (Polity)

(QUESTION PAPER)
DO NOT OPEN THIS TEST BOOKLET UNTIL YOU ARE TOLD TO DO SO
T.B.C.: SU-2024-pPLT Test Booklet Series

TEST BOOKLET
GENERAL STUDIES
pPLT-3.0 (Polity)
A
Time Allowed: 1 Hours

INSTRUCTIONS
D
Maximum Marks: 50

1. IMMEDIATELY AFTER THE COMMENCEMENT OF THE EXAMINATION, YOU SHOULD


CHECK THAT THIS TEST BOOKLET DOES NOT HAVE ANY UNPRINTED OR TORN OR
MISSING PAGES OR ITEMS, ETC. IF SO, GET IT REPLACED BY A COMPLETE TEST BOOKLET.
2. Please note that it is the candidate's responsibility to encode and fill in the Roll Number and Test
Booklet Series Code A, B, C or D carefully and without any omission or discrepancy at the
appropriate places in the OMR Answer Sheet. Any omission/discrepancy will render the Answer
Sheet liable for rejection.
3. You have to enter your Roll Number on the
Test Booklet in the Box provided alongside.
DO NOT write anything else on the Test Booklet.
4. This Test Booklet contains 50 items (questions). Each item is printed only in English. Each item comprises
four responses (answers). You will select the response you want to mark on the Answer sheet. In case you
feel that there is more than one correct response, mark the response which you consider the best. In any
case, choose ONLY ONE response for each item.
5. You have to mark all your responses ONLY on the separate Answer Sheet provided. See directions in the
Answer Sheet.
6. All items carry equal marks.
7. Before you proceed to mark in the Answer Sheet the response to various items in the Test Booklet, you
have to fill in some particulars in the Answer Sheet as per instructions sent to you with your Admission
Certificate.
8. After you have completed filling in all your response on the Answer Sheet and the examination has
concluded, you should hand over to the Invigilator only the Answer Sheet. You are permitted to take away
with you the Test Booklet.
9. Penalty for wrong answers:
THERE WILL BE PENALTY FOR WRONG ANSWERS MARKED BY A CANDIDATE IN THE
OBJECTIVE TYPE QUESTION PAPER.
(i) There are four alternatives for the answer to every question. For each question for which a wrong answer
has been given by the candidate, one-third of the marks assigned to that question will be deducted as
penalty.
(ii) If a candidate gives more than one answer, it will be treated as a wrong answer even if one of the given
answers happens to be correct and there will be same penalty as above to that question.
(iii)If a question is left blank, i.e., no answer is given by the candidate, there will be no penalty for that
question.
DO NOT OPEN THIS TEST BOOKLET UNTIL YOU ARE TOLD TO DO SO
MANTHAN-2024| PRELIMS TEST SERIES| pPLT-3.0
1. With regard to curative petition, consider 3. Which of the following conditions qualify a
the following statements: person to be appointed as judge of a High
1. It is filed after the review plea against
Court?
the final conviction is dismissed.
1. He is a citizen of India or Nepal.
2. It is mentioned under the Indian
constitution. 2. He has held a judicial office in the
3. It is accepted only when the bench of territory of India for ten years.
five senior-most judges approves it.
Select the correct answer using the code
How many of the statements given above
given below.
are correct?
(a) Only one (a) 1 only

(b) Only two (b) 2 only


(c) All three (c) Both 1 and 2
(d) None
(d) Neither 1 nor 2

2. Consider the following statements with


reference to Judge of the Supreme Court of 4. With reference to the Family Courts Act
India:
1984, consider the following statements:
1. The Constitution has not fixed the
1. The act makes it obligatory on the
tenure of the judge of the Supreme
Court. States Government to set up family

2. Any question regarding the age of the court in every city or town with a
judge of the Supreme Court shall be population exceeding ten lakhs.
determined by the President.
2. The appeal against the judgment of the
3. He can be removed by the president on
Family Court lies to the District court.
the recommendation of the judiciary
itself. Which of the statements given above is/are
How many of the statements given above correct?
are incorrect? (a) 1 only
(a) Only one
(b) 2 only
(b) Only two
(c) All three (c) Both 1 and 2
(d) None (d) Neither 1 nor 2

Copy is ENCRYPTED and for personal use ONLY| www.sunyaias.com |MANTHAN-2024 Page. 1
MANTHAN-2024| PRELIMS TEST SERIES| pPLT-3.0
5. Consider the following statements: 7. It is set up for the administration of a small
1. The power to increase the strength of
town. It is a semi-municipal authority and is
the judges of the Supreme Court lies
with the President. entrusted with a limited number of civic
2. While transferring the judge of the High
functions. It is created by a separate act of a
Courts the Chief justice of India should
consult with a collegium of two judges state legislature. Its composition, functions
of the Supreme Court only.
and other matters are governed by the act. It
3. The constitution provides that the
President can appoint a judge of the may be wholly elected or wholly nominated
Supreme Court as an acting Chief
Justice of India. by the state government or partly elected
How many of the statements given above and partly nominated. The above
are correct?
(a) Only one description refers to which of the following
(b) Only two types of urban local bodies?
(c) All three
(d) None (a) Notified Area Committee
(b) Town Area Committee
6. Consider the following statements with
reference to the power of Gram Sabha under (c) Township
the PESA (Provisions of the Panchayats
(Extension to Scheduled Areas) Act, 1996 (d) None
Act:
1. Every Gram Sabha shall be competent
to safeguard and preserve the traditions 8. With reference to the Rajya Sabha Forum on
and customs of the people, their cultural Panchayat Raj, consider the following
identity, community resources and the
customary mode of dispute resolution. statements:
2. Every Gram Sabha shall be responsible
1. Chairman of the Rajya Sabha is the
for the identification of beneficiaries
under the poverty alleviation and other President of the Forum.
programmes.
3. The recommendations of the Gram 2. The Forum shall consist of not more
Sabha or the Panchayats at the than 15 members of the Rajya Sabha.
appropriate level shall be mandatory for
grant of prospecting license or mining Which of the statements given above is/are
lease for major minerals in the correct?
Scheduled Areas. (a) 1 only
How many of the statements given above
(b) 2 only
are correct?
(a) Only one (c) Both 1 and 2
(b) Only two
(c) All three (d) Neither 1 nor 2
(d) None
Copy is ENCRYPTED and for personal use ONLY| www.sunyaias.com |MANTHAN-2024 Page. 2
MANTHAN-2024| PRELIMS TEST SERIES| pPLT-3.0
9. Consider the following statements with 11. With reference to the Metropolitan
reference to Central Council of Local Planning Committee under Part IX-A of the
Government: Constitution, consider the following
1. It was constituted by an act of the statements:
Parliament following the provisions
1. It prepares a draft development plan for
under Article 263 of the Indian
Constitution. the Metropolitan area as a whole.
2. It deals with both urban as well as rural 2. Two-thirds of the members of a
local governments. metropolitan planning committee
3. The Union Minister of Urban should be elected by the elected
Development acts as the chairman of members of the municipalities and
the council.
chairpersons of the panchayats in the
How many of the statements given above
metropolitan area from amongst
are correct?
themselves.
(a) Only one
(b) Only two Which of the statements given above is/are
(c) All three correct?
(d) None (a) 1 only
(b) 2 only
10. Consider the following statements with
(c) Both 1 and 2
reference to the election of members and
chairpersons mentioned 73rd Amendment (d) Neither 1 nor 2
Act of 1992
1. All the members of panchayats at the 12. With reference to the North-Eastern Council
village, intermediate and district levels (NEC), which of the following statements
shall be elected directly by the people.
is/ are correct?
2. The chairperson of panchayats at the
intermediate and district levels shall be 1. It comprises both Governors and Chief
elected indirectly by and from amongst Ministers of all North Eastern States as
the elected members thereof. its members.
3. The chairperson of a panchayat at the 2. The Union Minister of Home Affairs is
village level shall be elected directly by the Vice Chairman of NEC.
the people.
Select the correct answer given below:
How many of the statements given above
are correct? (a) 1 only
(a) Only one (b) 2 only
(b) Only two (c) Both 1 and 2
(c) All three (d) Neither 1 nor 2
(d) None
Copy is ENCRYPTED and for personal use ONLY| www.sunyaias.com |MANTHAN-2024 Page. 3
MANTHAN-2024| PRELIMS TEST SERIES| pPLT-3.0
13. Considers the following statement with 15. Which one of the following statements is
reference to Municipal Corporations: incorrect about the Panchayat Raj
1. They are established in the states by the Institution?
acts of the concerned state legislatures, (a) The Mandals/Blocks are to be
and in the union territories by the acts of mandatorily constituted in the smaller
the Parliament of India states.
2. A Municipal Corporation has usually (b) At the apex is the Zila Panchayat
three authorities, namely, the council, covering the entire rural area of the
the standing committees and the District.
commissioner.
(c) The Gram Sabha comprise of all the
3. All the councilors of the council are
adult members registered as voters in
directly elected by the people.
the Panchayat area.
How many of the statements given above
are correct? (d) If the Panchayat is dissolved before the
(a) Only one end of its five years’ term, fresh
(b) Only two elections must be held within six
(c) All three months of such dissolution.
(d) None

16. Consider the following statements:


14. Considers the following statement with
1. The district judge possesses original
reference to Cantonment board:
and appellate jurisdiction only in
1. It is set up as a statutory body.
criminal matters.
2. It consists of partly elected and partly
2. The district judge has supervisory
nominated members
powers over all the subordinate courts
3. The executive officer of the cantonment
in the district.
board is appointed by the Governor of
3. The session’s judge does not have the
the concerned state.
power to impose capital punishment.
How many of the statements given above
How many of the statements given above
are correct?
are incorrect?
(a) Only one (a) Only one
(b) Only two (b) Only two
(c) All three (c) All three
(d) None (d) None

Copy is ENCRYPTED and for personal use ONLY| www.sunyaias.com |MANTHAN-2024 Page. 4
MANTHAN-2024| PRELIMS TEST SERIES| pPLT-3.0
17. Reference to the Special Leave to appeal by 19. Consider the following statements:
the Supreme Court, consider the following 1. Under the Constitution (Seventy- Fourth
Amendment) Act, 1992, there is an
statements: optional provision for the constitution of
1. The Supreme Court is authorized to the Metropolitan Planning Committee in all
the metropolitan areas by the State
grant in its discretion special leave to
Governments.
appeal from any judgment in any matter 2. The Government of West Bengal was the
passed by any court or tribunal in the first to constitute the Kolkata Metropolitan
Planning Committee (KMPC) in the
country excluding military tribunal and
country.
court martial. 3. The Ministry of Home Affairs is the nodal
2. It is a discretionary power. ministry to oversee the implementation of
74th Constitution Amendment Act in the
Which of the statements given above is/are
states and the Union Territories.
correct? How many of the statements given above are
(a) 1 only correct?
(a) Only one
(b) 2 only (b) Only two
(c) Both 1 and 2 (c) All three
(d) None
(d) Neither 1 nor 2
20. What was the intention behind inclusion of
Articles 371 to 371-J in the Constitution of
18. Consider the following statements about
India?
Judicial Review: 1. To meet the aspirations of the people of
1. 'Judicial review' is explicitly stated in backward regions of the states
2. To deal with the disturbed law and order
the Constitution.
condition in some parts of the states
2. 'Judicial review' was borrowed concept 3. To deal with the border issues with
from Russia. neighboring countries in North-Western
India.
Which of the statements given above is/are
4. To protect the cultural and economic
correct? interests of the tribal people of the states
(a) 1 only How many of the statements given above are
correct?
(b) 2 only (a) Only one
(c) Both 1 and 2 (b) Only two
(c) Only three
(d) Neither 1 nor 2
(d) All four

Copy is ENCRYPTED and for personal use ONLY| www.sunyaias.com |MANTHAN-2024 Page. 5
MANTHAN-2024| PRELIMS TEST SERIES| pPLT-3.0
21. Consider the following statements with 23. Consider the following statements with
reference to the special provisions regarding reference to the District Planning
Manipur:
Committee (DPC) in India:
1. The President can create a committee of the
Manipur Legislative Assembly consisting 1. It consolidates the plans prepared by
of the members elected from the Hill Areas both panchayats and municipalities in
of the Manipur.
the district.
2. It is the responsibility of Parliament to
submit an annual report regarding the 2. The provisions of the composition of
administration of the hill areas of Manipur such committees are made by the
to the President.
Which of the statements given above is/are respective State legislature.
correct? 3. 30 per cent of the members of DPC are
(a) 1 only
nominated by the Governor of the
(b) 2 only
(c) Both 1 and 2 respective state.
(d) Neither 1 nor 2 How many of the statements given above
are incorrect?
22. Consider the following Committees and their (a) Only one
key recommendations with reference to
(b) Only two
Panchayati Raj Institutions (PRIs) in India:
Committee Recommendation (c) All three
1. Balwant Rai - Creation of Mandal (d) None
Mehta Panchayat
Committee
24. Which among the following are the sources
2. Ashok Mehta - Three-tier system
Committee- of income for the Urban Local Bodies
3. G.V.K. Rao - Creation of post of (ULBs) in India?
Committee District
1. Property Tax
Development
Commissioner 2. Payment for Public Utilities
4. L M Singhvi - Constitutional 3. Loan from financial institutions
Committee Recognition to
4. Excise Duty
Panchayats
How many of the above given pairs are How many of the statements given above
correctly matched? are correct?
(a) Only one (a) Only one
(b) Only two (b) Only two
(c) Only three (c) Only three
(d) All four
(d) All four

Copy is ENCRYPTED and for personal use ONLY| www.sunyaias.com |MANTHAN-2024 Page. 6
MANTHAN-2024| PRELIMS TEST SERIES| pPLT-3.0
25. With reference to local self-governance in 27. If a particular area is brought under the Fifth
India, consider the following statements: Schedule of the Constitution of India, which
1. The constitution does not confer any one of the following statements best reflects
inherent taxation powers on the local the consequence of it?
bodies. (a) This would prevent the transfer of land
2. The subject of Local governance falls of tribal people to non-tribal people.
under the Concurrent legislative
(b) This would create a local self-
jurisdiction under the Seventh Schedule
governing body in that area.
of the Constitution.
(c) This would convert that area into a
3. Powers and functions of Gram Sabha
are clearly mentioned in the Union Territory.
Constitution. (d) The State having such areas would be
How many of the statements given above declaring a Special Category State.
are correct?
(a) Only one 28. With reference to the lawsuits by or against
(b) Only two the government, consider the following
(c) All three
statements:
(d) None
1. While filing a court case against
government, the Central government
26. Which of the following are the mandatory
should be referred to as the
provisions under the 73rd Constitutional
Amendment Act, 1992? ‘Government of India’.

1. Constitution of a State Finance 2. A lawsuit can be filed against the


Commission after every five years. government by an organization for
2. Indirect elections to the post of breaching of contractual liabilities.
chairperson of panchayats at the district 3. The Government in India can be sued
levels for civil wrongs committed by its
3. Giving representation to members of officials while performing the non-
the Parliament in the panchayats sovereign functions.
How many of the statements given above How many of the statements given above
are correct? are correct?
(a) Only one (a) Only one
(b) Only two (b) Only two
(c) All three (c) All three
(d) None (d) None

Copy is ENCRYPTED and for personal use ONLY| www.sunyaias.com |MANTHAN-2024 Page. 7
MANTHAN-2024| PRELIMS TEST SERIES| pPLT-3.0
29. Consider the following statements with 31. With reference to the Fifth Schedule of the
reference to the Model Panchayat Citizen Constitution, consider the following
Charter framework: statements:
1. It has been released by Ministry of 1. A region is declared as a Scheduled
Rural Development in collaboration Area by the President but any change in
with National Institute of Rural its area can be done only by the
Development. Governor of respective State.
2. It aims to align the delivery of public 2. The executive power of a State does not
services with Sustainable Development extend to the Scheduled Areas.
Goals at panchayat level. 3. The President can direct to establish a
3. Under this framework, draft of Citizen Tribes Advisory Council in states
without Scheduled Areas.
Charters for panchayats needs the
4. A Governor can make a regulation to
approval of the respective Gram Sabha.
repeal any act of the state legislature in
How many of the statements given above Scheduled Areas, with President’s
are correct? assent.
(a) Only one How many of the statements given above
(b) Only two are correct?
(c) All three (a) Only one
(d) None (b) Only two
(c) Only three
30. With reference to the ‘District Councils’ for (d) All four
Tribal Areas of some states, consider the
following statements: 32. The Indian Constitution prescribes
1. Each Autonomous district has a fully mandatory reservation for Scheduled Caste
elected district council. and Scheduled Tribes in which of the
2. Laws made on village administration by following bodies?
these Councils require the assent of the 1. Lok Sabha
Governor. 2. Rajya Sabha
3. The Governor is empowered to dissolve 3. Legislative assembly of a state
the district council on recommendation 4. All three levels of panchayat bodies
of the State Government. 5. Chairpersons of Urban Local Bodies
How many of the statements given above How many of the statements given above
are correct? are correct?
(a) Only two
(a) Only one
(b) Only three
(b) Only two
(c) Only four
(c) All three
(d) All five
(d) None

Copy is ENCRYPTED and for personal use ONLY| www.sunyaias.com |MANTHAN-2024 Page. 8
MANTHAN-2024| PRELIMS TEST SERIES| pPLT-3.0
33. Consider the following statements with 35. Which of the following statements is
reference to the ‘Control of the Union over incorrect regarding Public Interest
the Administration of Scheduled Areas and Litigation.
the Welfare of Scheduled Tribes’: 1. Any citizen can file PIL only in
Supreme Court and High court
1. According to the Constitution, the
2. A PIL may be filed against state
Parliament must appoint a Commission
government, central government,
to report on the administration of the municipal authority but not against
Scheduled Areas. private party.
2. Union has executive power to give 3. PIL is not defined in any statute or in
directions to a State for the welfare of any act.
the Scheduled Tribes in the State. How many of the statements given above
are incorrect?
3. First commission regarding the
(a) Only one
administration of Scheduled Areas and (b) Only two
the Welfare of STs was appointed under (c) All three
DB Kalelkar. (d) None
How many of the statements given above
are correct? 36. With reference to the Indian judiciary,
consider the following statements:
(a) Only one
1. Chief Justice can himself decides to
(b) Only two
open the bench of Supreme Court
(c) All three
anywhere in India.
(d) None
2. A High Court in India has the power to
review its own judgment as the
34. For which among the following States, there Supreme Court does.
are the provisions for the administration of 3. Any retired judge of the Supreme Court
Tribal Areas under the Sixth Schedule of the of India can be called back to sit and act
Constitution? as a Supreme Court judge by the Chief
Justice of India with the prior
(a) Arunachal Pradesh, Mizoram, Tripura,
permission of the President of India.
Manipur
How many of the statements given above
(b) Assam, Manipur, Tripura, Mizoram are correct?
(c) Assam, Meghalaya, Tripura, Mizoram (a) Only one
(d) Arunachal Pradesh, Meghalaya, (b) Only two
Tripura, Mizoram (c) All three
(d) None

Copy is ENCRYPTED and for personal use ONLY| www.sunyaias.com |MANTHAN-2024 Page. 9
MANTHAN-2024| PRELIMS TEST SERIES| pPLT-3.0
37. Consider the following statements in 39. Which of the following are the positive
regards to Benches of Supreme Court impacts of 'judicial activism':
1. A Constitution Bench is a bench of the 1. The idea of rights gets expanded.
Supreme Court having at least seven or 2. Democratisation of the judicial system.
more judges on it.
3. Blurred the line of distinction between
2. Presently, Constitution Benches are set
the legislature, executive and judiciary.
up on an ad hoc basis as and when the
How many of the statements given above
need arises. are incorrect?
3. A bench of two judges is called a (a) Only one
division bench and a bench of three or (b) Only two
five judges is called full bench. (c) All three
How many of the statements given above (d) None
are correct?
(a) Only one
(b) Only two 40. Consider the following pairs:
(c) All three 1. The Doctrine of : Federalism
(d) None Harmonious
Construction
38. In context of Review petition, examine the 2. The Doctrine of : The Lists of
following statements: Pith and Substance the Seventh
1. As per the Civil Procedure Code and the Schedule
Supreme Court Rules, only aggrieved 3. The Doctrine of : Limits the
person by a ruling can seek a review Colourable legislature
within 1 week of judgment or order Legislation capacity to
pronounced. enact laws
2. The court has the power to review its which are
rulings to correct both "patent error" outside its
and "minor mistakes of inconsequential
competence
import.
How many of the statements given above
Select the correct answer code: are correct?
(a) 1 only (a) Only one
(b) 2 only (b) Only two
(c) Both 1 and 2 (c) All three
(d) Neither 1 nor 2 (d) None

Copy is ENCRYPTED and for personal use ONLY| www.sunyaias.com |MANTHAN-2024 Page. 10
MANTHAN-2024| PRELIMS TEST SERIES| pPLT-3.0
41. Which of the following are the essential 43. Which of the following statements are
aspects of natural justice: correct regarding Administrative Tribunals?
1. Right to fair hearing. 1. In pursuance of the Article 323A, the
2. Judges recusing themselves in case of Parliament has passed the
vested interests. Administrative Tribunals Act in1985.
3. Judgments given by the courts should 2. There will be a Chairman, Vice-
be well-reasoned. Chairman and 25 members in the
4. Judicial review of the laws. Central Administrative Tribunal along
5. Access to cheap justice. its branches.
How many of the statements given above 3. The Chairman will be appointed by the
are correct? Ministry of Home, GOI
(a) Only two 4. The members of the State
(b) Only three Administrative Tribunals are appointed
(c) Only four
by the President after consultation with
(d) All five
the concerned State Chief Minister’s.
How many of the statements given above
42. Consider the following statements regarding are incorrect?
tribunals in India: (a) Only one
1. The word Tribunals" is nowhere (b) Only two
mentioned in the Indian Constitution. (c) Only three
2. Tribunals under Article 323 can only be (d) All four
established by Parliament unlike 323A
which can also be established by State 44. In context of Anticipatory bail in India,
Legislature examine the following statements:
1. It can be obtained by a person who
3. There is no question of the hierarchy of
anticipates arrest.
tribunals under Article 323 A whereas
in 323 B hierarchy of tribunals may be 2. It is issued only by the Supreme Court
created. and High Court.
How many of the statements given above Select the correct answer code:
are correct? (a) 1 only
(a) Only one (b) 2 only
(b) Only two (c) Both 1 & 2
(c) All three (d) Neither 1 nor 2
(d) None

Copy is ENCRYPTED and for personal use ONLY| www.sunyaias.com |MANTHAN-2024 Page. 11
MANTHAN-2024| PRELIMS TEST SERIES| pPLT-3.0
45. Consider the following statements regarding 47. Consider the following statements regarding
‘judicial restraint’:
the Supreme court of India:
1. It is against the concept of balance of
1. The Constitution authorizes the power.
President to appoint places other than 2. It violates the independence of the
Judiciary.
Delhi as seat of the Supreme Court.
3. It recognizes the equality of the other two
2. All proceedings in the Supreme court branches with the judiciary.
should be in English, but petitions can How many of the statements given above are
correct?
be allowed in Hindi along with English. (a) Only one
Which of the statements given above is/are (b) Only two
(c) All three
correct? (d) None
(a) 1 only
48. Consider the following statements:
(b) 2 only
1. The motion to impeach a Judge of the
(c) Both 1 and 2 Supreme Court of Indian cannot be rejected
by the Speaker of the Lok Sabha as per the
(d) Neither 1 nor 2
Judges (Inquiry) Act, 1968.
2. The Constitution of India defines and gives
details of what constitutes ‘incapacity and
46. With reference to the ‘Gram Nyayalaya
proved misbehaviour’ of the Judges of the
Act’, which of the following statements Supreme Court of India.
is/are correct? 3. The process of impeachment of the Judges
of the Supreme Court of India is given in
1. As per the Act, Gram Nyayalayas can the Judges (Inquiry) Act, 1968.
hear only civil cases and not criminal 4. If the motion for the impeachment of a
Judge is taken up for voting, the law
cases.
requires the motion to be backed by each
2. The Act allows local social activists as House of the Parliament and supported by
mediators/ reconciliators. a majority of total membership of that
House and by not less than two-thirds of
Select the correct answer using the code total members of that House present and
given below. voting.
How many of the statements given above are
(a) 1 only correct?
(b) 2 only (a) Only one
(b) Only two
(c) Both 1 and 2 (c) Only three
(d) Neither 1 nor 2 (d) All four

Copy is ENCRYPTED and for personal use ONLY| www.sunyaias.com |MANTHAN-2024 Page. 12
MANTHAN-2024| PRELIMS TEST SERIES| pPLT-3.0
49. With reference to Lok Adalat, which of the 50. Which of the following is/are regarded as
following statements is correct? Criminal Contempt of Court?
1. When Judicial acts are criticized in a
(a) Lok Adalat have the jurisdiction to
newspaper even in a fair and reasonable
settle the matters at pre-litigative stage way.
and not those matters pending before 2. An act which interferes with due course
any court. of judicial proceedings.
3. Commenting on administrative side of
(b) Lok Adalat deals with matters which
Judiciary.
are civil and not criminal in nature.
4. Innocent publication and discussion of
(c) Every Lok Adalat consists of either matters of court.
serving or retired judicial officers only. 5. Willful disobedience of any judgment
of court.
(d) The Legal Services Authorities Act,
How many of the statements given above
1987 provides for the establishment of
are incorrect?
Permanent Lok Adalat. (a) Only two
(b) Only three
(c) Only four
(d) All five

******

Copy is ENCRYPTED and for personal use ONLY| www.sunyaias.com |MANTHAN-2024 Page. 13
MANTHAN-2024| PRELIMS TEST SERIES| pPLT-3.0

Copy is ENCRYPTED and for personal use ONLY| www.sunyaias.com |MANTHAN-2024 Page. 2
MANTHAN
PRELIMS TEST SERIES-2024

pPLT-3.0 (Polity)

(SOLUTIONS & EXPLANATIONS)


MANTHAN-2024 | PRELIMS TEST SERIES | pPLT-3.0
(SOLUTIONS & EXPLANATIONS)
1. Answer: (A) Only one statement is correct.
Explanation:
● Statement 1 is correct: About Curative Petition: The concept of the curative petition first evolved by the
Supreme Court in Rupa Ashok Hurra vs. Ashok Hurra & another case (2002) on the question whether an
aggrieved person is entitled to any relief against the final judgment/order of the Supreme Court, even after the
dismissal of a review petition.
● Statement 2 is incorrect: It is based on interpretation of Article 137, which provides that in the matter of
laws and rules made under Article 145, Supreme Court has the power to review any judgment pronounced (or
order made) by it. Thus, it is not mentioned in the question directly. Its objective is to avoid miscarriage of
justice and to prevent abuse of process.
Procedure
● Statement 3 is incorrect: A curative petition may be filed after a review plea against the final conviction is
dismissed. It can be entertained if the petitioner establishes that there was a violation of the principles of
natural justice, and that he was not heard by the court before passing an order.
⮚ A curative petition must be first circulated to a Bench of the three senior-most judges, and the judges
who passed the concerned judgment, if available. Only when a majority of the judges conclude that the
matter needs hearing should it be listed before the same Bench.
⮚ A curative petition is usually decided by judges in the chamber unless a specific request for an open-
court hearing is allowed.
⮚ In the event of the Bench holding at any stage that the petition is without any merit, it may impose a
penalty on the petitioner.

2. Answer: (B) Only two statements are incorrect.


Explanation:
● Statement 1 is correct: The Constitution has not fixed the tenure of a judge of the Supreme Court.
● Statement 2 is incorrect: He holds office until he attains the age of 65 years. Any question regarding his age
is to be determined by such authority and in such manner as provided by Parliament.
● Statement 3 is incorrect: He can resign his office by writing to the President.
⮚ He can be removed from his office by the President on the recommendation of the Parliament.
⮚ The President can issue the removal order only after an address by Parliament has been presented to him
in the same session for such removal.
⮚ The address must be supported by a special majority of each House of Parliament (i.e., a majority of the
total membership of that House and a majority of not less than two-thirds of the members of that House
present and voting). The grounds of removal are two—proved misbehavior or incapacity.

3. Answer: B
Explanation:
Qualifications to Become a High Court Judge: A person to be appointed as a judge of a high court should have
following qualifications:
● Statement 1 is incorrect: He should be a citizen of India;
● Statement 2 is correct: He should have held a judicial office in the territory of India for ten years or
⮚ He should have been an advocate of high court(s) for ten years.
⮚ There is no minimum age fixed for high Court judges, and unlike in Supreme Court, there is no provision
⮚ For the appointment of a distinguished jurist as a judge of a high court.

4. Answer: A
Explanation:
⮚ The Family Courts Act, 1984 provides for establishment of Family Courts by the State Governments in
consultation with the High Court’s with a view to promote conciliation and secure speedy settlement of
disputes relating to marriage and family affairs.
● Statement 1 is correct: It is mandatory for the State Government to set up a Family Court for every area in
the State comprising a city or a town whose population exceeds one million.
Copy is ENCRYPTED and for personal use ONLY| www.sunyaias.com |MANTHAN-2024 Page. 2
MANTHAN-2024 | PRELIMS TEST SERIES | pPLT-3.0
Features of the Act:
⮚ The Family Courts are free to evolve their own rules of procedure, which override the rules of procedure
contemplated under the Code of Civil Procedure.
⮚ Special emphasis is put on settling the disputes by mediation and conciliation, when the matter is solved
by an agreement between both the parties; it reduces the chances of any further conflict.
⮚ The cases are kept away from the trappings of a formal legal system, which can be a very traumatic
experience for the families and lead to personal and financial losses that can have a devastating effect on
human relations as well.
⮚ A Family court has jurisdiction in both civil and criminal matters under the act.
⮚ Under civil matters, it can deal with any suit and proceeding related to matrimonial issues, spousal
property, the legitimacy of any person, maintenance, guardianship of the person or custody of or access
to any minor.
⮚ Under criminal matters, it has jurisdiction over orders related to maintenance of wife, children and
parents described under Chapter IX of Criminal Procedure Code (Cr. P. C.).
⮚ The act provides that parties to a dispute before a Family court shall not be entitled right to be represented
by some legal practitioners. However, the court in the interest of justice seek the assistance of a legal
practitioner as amicus curiae.
⮚ Family Court is free to consider any evidence if it assists effectively in dealing with a dispute despite its
admissibility under the Indian Evidence Act, 1872.
⮚ The proceedings before the Family Court are first referred to conciliation and only when the conciliation
proceedings fail to resolve the issue successfully, the matter taken up for trial by the Court. The act
provides for the social welfare agencies, counselors during the conciliation stage and to secure the service
of medical and welfare experts.
● Statement 2 is incorrect: An appeal against the judgment or order passed by the Family Court can be filed
in High Court preferably within 30 Days.

5. Answer: (A) Only one statement is correct.


Explanation:
● Statement 1 is incorrect: Few years ago, Parliament passed the legislation to increase the sanctioned strength
of the Supreme Court from 31 to 34 including the Chief Justice of India.
⮚ Constitutional provisions Originally, under Article 124 of the Indian Constitution the strength of
Supreme Court was fixed at eight (one chief justice and seven other judges).
⮚ Article 124 (1) provides the power to the Parliament to increase the number of judges if it deems
necessary.
⮚ The Parliament through The Supreme Court (Number of Judges) Act, 1956 increased strength of
Supreme Court to ten.
⮚ At present, the Supreme Court consists of thirty-one judges (one chief justice and thirty other judges).
Transfer of Judges from one High Court to another High Court:
It is made by the President after consultations with the Chief Justice of India under Article 222 (1) of the
Constitution.
The Supreme Court derives its power to select, appoint and transfer judges from its verdicts in Three Judges
Cases.
● Statement 2 is incorrect: CJI should consult with four senior-most Supreme Court judges and the chief
justice of two high courts (one from which the judge is being transferred and the other receiving him) to form
his opinion on judicial appointments and transfers.
● Statement 3 is correct: Article 223 and 126 states that the President can appoint a judge of a high court as
acting chief justice of the high court when:
⮚ The office of chief justice of the high court (or the Supreme Court) is vacant; or
⮚ The chief justice of the high court (or the Supreme Court) is temporarily absent; or
⮚ The chief justice of the high court (or the Supreme Court) is unable to perform the duties of his office.

Copy is ENCRYPTED and for personal use ONLY| www.sunyaias.com |MANTHAN-2024 Page. 3
MANTHAN-2024 | PRELIMS TEST SERIES | pPLT-3.0
6. Answer: (B) Only two statements are correct.
Explanation:
● Statement 1 is correct: The features (or the provisions) of the PESA Act are as follows:
⮚ State legislation on the Panchayat in the Scheduled Areas shall be in consonance with the customary
law, social and religious practices and traditional management practices of community resources.
⮚ A village shall ordinarily consist of a habitation or a group of habitations or a hamlet or a group of
hamlets comprising a community and managing its affairs in accordance with traditions and customs.
⮚ Every village shall have a Gram Sabha consisting of persons whose names are included in the electoral
rolls for the Panchayat at the village level.
⮚ Every Gram Sabha shall be competent to safeguard and preserve the traditions and customs of the people,
their cultural identity, community resources and the customary mode of dispute resolution.
● Statement 2 is correct: Every Gram Sabha shall—
⮚ Approve of the plans, programmes and projects for social and economic development before they are
taken up for implementation by the Panchayat at the village level; and
⮚ Be responsible for the identification of beneficiaries under the poverty alleviation and other programmes.
⮚ Every Panchayat at the village level shall be required to obtain from the Gram Sabha a certification of
the utilization of funds for the above plans, programmes and projects.
⮚ The reservation of seats in the Scheduled Areas in every Panchayat shall be in proportion to the
⮚ Population of the communities for whom the reservation is sought to be given under Part IX of the
Constitution.
⮚ However, the reservation for the Scheduled Tribes shall not be less than one-half of the total number of
seats. Further, all seats of Chairpersons of Panchayats at all levels shall be reserved for the Scheduled
Tribes.
● Statement 3 is incorrect: The recommendations of the Gram Sabha or the Panchayats at the appropriate level
shall be mandatory for grant of prospecting license or mining lease for minor minerals (not major) in the
Scheduled Areas.

7. Answer: B
Explanation:
● A Town Area Committee is set up for the administration of a small town. It is a semi-municipal authority
and is entrusted with a limited number of civic functions like drainage, roads, street lighting, and conservancy.
It is created by a separate act of a state legislature. Its composition, functions and other matters are governed
by the act. It may be wholly elected or wholly nominated by the state government or partly elected and partly
nominated. Hence option (b) is the correct answer.
● Notified Area Committee: A notified area committee is created for the administration of two types of areas-
a fast developing town due to industrialization, and a town which does not yet fulfill all the conditions
necessary for the constitution of a municipality, but which otherwise is considered important by the state
government. It is called so because it is created by notification and unlike the municipality, it is an entirely
nominated body, i.e. all members, including the Chairman, are nominated by the state government. Thus, it is
neither a statutory body (created by law) nor an elected body.
● Township: This type of urban government is established by the large public enterprises to provide civic
amenities to its staff and workers who live in the housing colonies built near the plant. The enterprise appoints
a town administrator to look after the administration of the township. It has no elected members.
● Port Trust: The port trusts are established in the port areas like Mumbai, Kolkata, Chennai and so on for two
purposes: (a) to manage and protect the ports; and (b) to provide civic amenities. A port trust is created by an
Act of Parliament. It consists of both elected and nominated members. Its chairman is an official. Its civic
functions are more or less similar to those of a municipality.

Copy is ENCRYPTED and for personal use ONLY| www.sunyaias.com |MANTHAN-2024 Page. 4
MANTHAN-2024 | PRELIMS TEST SERIES | pPLT-3.0
8. Answer: B
Explanation:
● Statement 1 is incorrect: The Hon'ble Chairman, Rajya Sabha constituted the Rajya Sabha Forum on
Panchayati Raj on 6th February 2016 with the Hon'ble Deputy Chairman, Rajya Sabha, as the ex-officio
President of the Forum and the Minister of Panchayati Raj as the ex-officio Vice-President of the Forum.
● Statement 2 is correct: The Forum shall consist of not more than 15 members of Rajya Sabha, to be
nominated by the Hon'ble Chairman, Rajya Sabha, from amongst members who have special knowledge or
keen interest in the subject. The term of the members in the Forum shall be co-terminus with their membership
of the Rajya Sabha. Besides, experts in the field of Panchayati Raj may be associated as Special Invitees who
may share their views/present papers during the meetings of the Forum.
Objectives of the Rajya Sabha Forum on Panchayati Raj:
The Forum shall-
⮚ Identify problems and have focused deliberations relating to Panchayati Raj Institutions (PRIs);
⮚ Discuss strategies to strengthen PRIs; and
⮚ Provide a platform to Parliamentarians for exchange of ideas, views, experiences, expertise and Best
practices in relation to functioning and strengthening of PRIs.
⮚ The Forum will not interfere with or encroach upon the jurisdiction of the concerned
Ministry/Department or Department related Parliamentary Standing Committee.

9. Answer: (A) Only one statement is correct.


Explanation:
● Statement 1 is incorrect: The Central Council of Local Government was set up in 1954. It was constituted
under Article 263 of the Constitution of India by an order of the President of India.
● Statement 2 is incorrect: Originally, it was known as the Central Council of Local Self-Government.
However, the term ‘self- government’ was found to be superfluous and hence was replaced by the term
‘government’ in the1980s. Till 1958, it dealt with both urban as well as rural local governments, but after
1958 it has been dealing with matters of urban local government only.
● Statement 3 is correct: The Council is an advisory body. It consists of the Minister for Urban Development
in the Government of India and the ministers for local self-government in states. The Union Minister of Urban
Development acts as the chairman of the council.
The Council performs the following functions with regard to local government:
⮚ Considering and recommending the policy matter
⮚ Making proposals for legislation
⮚ Examining the possibility of cooperation between the Centre and the states
⮚ Drawing up a common programme of action
⮚ Recommending Central financial assistance
⮚ Reviewing the work done by the local bodies with the Central financial assistance

10. Answer: (B) Only two statements are correct.


Explanation:
Features of 73rd Constitutional Amendment Act, 1992 related to Election of Members and Chairpersons:
● Statement 1 is correct: All the members of panchayats at the village, intermediate and district levels shall be
elected directly by the people.
● Statement 2 is correct: The chairperson of panchayats at the intermediate and district levels shall be elected
indirectly—by and from amongst the elected members thereof.
● Statement 3 is incorrect: However, the chairperson of a panchayat at the village level shall be elected in
such manner as the state legislature determines.

Copy is ENCRYPTED and for personal use ONLY| www.sunyaias.com |MANTHAN-2024 Page. 5
MANTHAN-2024 | PRELIMS TEST SERIES | pPLT-3.0
ADDITIONAL INFORMATION:
Composition of Panchayats. -
Subject to the provisions of this Part, the Legislature of a State may, by law, make provisions with respect to the
composition of Panchayats:
⮚ Provided that the ratio between the population of the territorial area of a Panchayat at any level and the number
of seats in such Panchayat to be filled by election shall, so far as practicable, be the same throughout the State.
⮚ All the seats in a Panchayat shall be filled by persons chosen by direct election from territorial constituencies
in the Panchayat area and; for this purpose, each Panchayat area shall be divided into territorial constituencies
in such manner that the ratio between the population of each constituency and the number of seats allotted to
it shall, so far as practicable, be the same throughout the Panchayat area.

11. Answer: C
Explanation:
● Statement 1 is correct: Every metropolitan area shall have a metropolitan planning committee to prepare a
draft development plan for the Metropolitan area.
The state legislature may make provisions with respect to the following:
⮚ The composition of such committees;
⮚ The manner of election of members of such committees;
⮚ The representation in such committees of the Central government, state government and other
organizations.
⮚ The functions of such committees in relation to planning and coordination for the metropolitan area;
⮚ The manner of election of chairpersons of such committees.
● Statement 2 is correct: The Act lays down that two-thirds of the members of a metropolitan planning
committee should be elected by the elected members of the municipalities and chairpersons of the panchayats
in the metropolitan area from amongst themselves. The representation of these members in the committee
should be in proportion to the ratio between the population of the municipalities and the panchayats in that
metropolitan area.

12. Answer: A
Explanation:
● Statement 1 is correct: North Eastern Council (NEC)NEC was constituted as a statutory body under the NEC
Act 1971. It was constituted for securing the balanced development of North Eastern Areas comprising of
seven states -Arunachal Pradesh, Assam, Nagaland, Manipur, Meghalaya, Mizoram, Tripura. The NEC Act
1971 was amended in 2002. The 'North-Eastern Areas' now means the area comprising the eight states of
Assam, Manipur, Meghalaya, Nagaland, Tripura, Arunachal Pradesh, Sikkim, and Mizoram. Thus NEC
comprises eight states. The Council comprises of Governors and Chief Ministers of constituent States and
three members to be nominated by the President as per Clause (iii) of Section 3 of the North Eastern Council
(Amendment) Act, 1971.The North Eastern Council is the nodal agency for the economic and social
development of the North Eastern Region,
Composition
● Statement 2 is incorrect: Chairperson -Union Home Minister (ex-officio)
Vice Chairperson -Minister of State (Independent Charge) Ministry of Development of North Eastern Region
(DoNER)
Members: Governor and Chief Minister of eight states.

13. Answer: (B) Only two Statements are correct.


Explanation:
● Statement 1 is correct: Municipal corporations are created for the administration of big cities like Delhi,
Mumbai, Kolkata, Hyderabad, Bangalore and others. They are established in the states by the acts of the
concerned state legislatures, and in the union territories by the acts of the Parliament of India.

Copy is ENCRYPTED and for personal use ONLY| www.sunyaias.com |MANTHAN-2024 Page. 6
MANTHAN-2024 | PRELIMS TEST SERIES | pPLT-3.0
● Statement 2 is correct: A municipal corporation has three authorities, namely, the council, the standing
committees and the commissioner. Hence
● Statement 3 is incorrect: The Council is the deliberative and legislative wing of the corporation. It consists
of the Councilors directly elected by the people, as well as a few nominated persons having knowledge or
experience of municipal administration.

14. Answer: (B) Only two Statements are correct.


Explanation:
● Statement 1 is correct: A cantonment board is established for municipal administration for the civilian
population in the cantonment area. It is set up under the provisions of the Cantonments Act of 2006—
legislation enacted by the Central government.
● Statement 2 is correct: It works under the administrative control of the defense ministry of the Central
government. Thus, unlike the above four types of urban local bodies, which are created and administered by
the state government, a cantonment board is created as well as administered by the Central government. A
cantonment board consists of partly elected and partly nominated members. The elected members hold office
for a term of five years while the nominated members (i.e., ex-officio members) continue so long as they hold
the office in that station.
● Statement 3 is incorrect: The executive officer of the cantonment board is appointed by the president of
India. He implements all the resolutions and decisions of the board and its committees. He belongs to the
central cadre established for the purpose.

15. Answer: A
Explanation:
● All States now have a uniform three tier Panchayati Raj structure: At the base is the ‘Gram Panchayat’. A
Gram Panchayat covers a village or group of villages.
● Option A is incorrect: The intermediary level is the Mandal (also referred to as Block or Taluka). These
bodies are called Mandal or Taluka Panchayats. The intermediary level body need not be constituted in smaller
States.
● Option B is correct: At the apex is the Zila Panchayat covering the entire rural area of the District.
● Option C is correct: The amendment also made a provision for the mandatory creation of the Gram Sabha.
The Gram Sabha would comprise of all the adult members registered as voters in the Panchayat area. Its role
and functions are decided by State legislation.
● Option D is correct: The term of each Panchayat body is five years. If the State government dissolves the
Panchayat before the end of its five-year term, fresh elections must be held within six months of such
dissolution. This is an important provision that ensures the existence of elected local bodies. Before the 73 rd
amendment, in many States, there used to be indirect elections to the district bodies and there was no provision
for immediate elections after dissolution.

16. Answer: (B) Only two Statements are incorrect.


Explanation:
● Statement 1 is incorrect: The district judge is the highest judicial authority in the district. He possesses
original and appellate jurisdiction in both civil as well as criminal matters. In other words, the district judge
is also the session’s judge. When he deals with civil cases, he is known as the district judge and when he hears
the criminal cases, he is called as the session’s judge.
● Statement 2 is correct: The district judge exercises both judicial and administrative powers. He also has
supervisory powers over all the subordinate courts in the district. Appeals against his orders and judgments
lie to the High Court.

Copy is ENCRYPTED and for personal use ONLY| www.sunyaias.com |MANTHAN-2024 Page. 7
MANTHAN-2024 | PRELIMS TEST SERIES | pPLT-3.0
● Statement 3 is incorrect: The session’s judge has the power to impose any sentence including life
imprisonment and capital punishment (death sentence). However, a capital punishment passed by him is
subject to confirmation by the High Court, whether there is an appeal or not. Below the District and Sessions
Court stands the Court of Subordinate Judge on the civil side and the Court of Chief Judicial Magistrate on
the criminal side. The subordinate judge exercises unlimited pecuniary jurisdiction over civil suits. The chief
judicial magistrate decides criminal cases which are punishable with imprisonment for a term up to seven
years.

17. Answer: C
Explanation:
● Statement 1 is correct: The Supreme Court is authorized to grant in its discretion special leave to appeal
from any judgment in any matter passed by any court or tribunal in the country (except military tribunal and
court martial).
This provision contains the four aspects as under:
● Statement 2 is correct: It is a discretionary power and hence, cannot be claimed as a matter of right.
● It can be granted in any judgment whether final or interlocutory.
● It may be related to any matter—constitutional, civil, criminal, income tax, labour, revenue advocates, etc.
● It can be granted against any court or tribunal and not necessarily against a high court (except a military court).
What is a Special Leave Petition (SLP)?
● Generally, an appeal to the Supreme Court is made after an unsatisfactory judgment by the High Court, but
under certain special circumstances, an appeal may lie directly to the Supreme Court from any Court in India
under Article 136 of the Constitution. According to Article 136, an appeal can be made to the Supreme Court
directly from any order, decision, decree, judgment, etc given by any court or tribunal in India.

18. Answer: D
Explanation:
● Statement 1 is incorrect: Judicial Review: The phrase 'judicial review' is not explicitly provided in the
Constitution. However, the judiciary draws the power of judicial review from various provisions of the
Constitution (e.g., Articles 13, 32, 131, etc.).
● Statement 2 is incorrect: The doctrine of judicial review originated and developed in the USA. It was
propounded by John Marshall, the then chief justice of the American Supreme Court, for the first time in the
famous case of Marbury V. Madison (1803)
WHAT IS JUDICAL REVIEW?
● Judicial review is the power of the judiciary to examine the constitutionality of executive orders and legislative
enactments of both the State and Central governments. If, after this examination, they are found to be violative
of the Constitution, they can be declared as unconstitutional, illegal, and invalid (null and void) by the
judiciary. The power of judicial review is itself conferred by the Constitution on the judiciary (both the
Supreme Court as well as High Courts) in India. Furthermore, the judiciary has declared the power of judicial
review as a basic feature of the constitution or an element of the basic structure of the Constitution. Hence,
even by a constitutional amendment, the power of judicial review cannot be excluded or curtailed.
● On three grounds, the Constitutional validity of an executive order or a legislative enactment can be
challenged in the Supreme Court or in the High Court. These are: firstly, it infringes the Fundamental Rights;
secondly, it is outside the competence of the authority which has framed it, and lastly, it is repugnant to the
constitutional provisions. This clearly shows how the scope of judicial review in India is narrower than what
exists in the USA. This is because the American system provides for ‘due process of law’ whereas the Indian
Constitution provides for ‘procedure established by law’. The basic difference between these two is that in
‘due process of law’ the reasonableness, policy implications, or suitability of law is kept in mind whereas in
‘procedure established by law’ the law is examined substantively without taking into account the
reasonableness, policy implications, or suitability.

Copy is ENCRYPTED and for personal use ONLY| www.sunyaias.com |MANTHAN-2024 Page. 8
MANTHAN-2024 | PRELIMS TEST SERIES | pPLT-3.0

19. Answer: (A) Only one statement is correct.


Explanation:
● Statement 1 is incorrect: Under the Constitution (Seventy- Fourth Amendment) Act, 1992, there is a
mandatory provision for the constitution of the Metropolitan Planning Committee in all the metropolitan
areas by the State Governments. As per the Constitution Amendment Act, the Metropolitan Area‖ means an
area having a population of 10 lakh or more, comprised in one or more districts and consisting of two or more
municipalities or Panchayats or other contiguous areas specified and notified by the State Government to be
a Metropolitan Area for this purpose.
● Statement 2 is correct: The Government of West Bengal was the first to constitute the Kolkata Metropolitan
Planning Committee (KMPC) in the country.
● Statement 3 is incorrect: The Ministry of Housing and Urban Affairs is the nodal ministry to oversee the
implementation of 74th Constitution Amendment Act in the states and the Union Territories.

20. Answer: (C) Only three statements are correct.


Explanation:
● Statements 1, 2 and 4 are correct: Articles 371 to 371-J in Part XXI of the constitution contain special
provisions for twelve states viz., Maharashtra, Gujarat, Nagaland, Assam, Manipur, Andhra Pradesh,
Telangana, Sikkim, Mizoram, Arunachal Pradesh, Goa and Karnataka. Originally, the constitution did not
make any special provisions for these states. They have been incorporated by the various subsequent
amendments made in the context of reorganization of the states or conferment of statehood on the Union
Territories. The intention behind adding special provisions to the constitution are following:
I. To meet the aspirations of the people of backward regions of the states
II. To protect the cultural and economic interests of the tribal people of the states
III. To deal with the disturbed law and order condition in some parts of the states
IV. To protect the interests of the local people of the states.
Hence statement 1, 2 and 4 is correct.
● Statement 3 is incorrect: To deal with border issues in North-Western India is not one of the reasons behind
providing special provisions to these states. If we see the states that are provided for the special provisions
under the part, only a part of Gujarat can be considered as part of North-Western India. But in Gujarat also
the rationale behind providing such special provision is to ensure equitable development throughout the state,
not to deal with border issue.

21. Answer: A
Explanation:
● Statement 1 is correct: Article 371-C makes the special provisions for Manipur to meet the aspirations of
the people of backward regions of the states. Under article 371-C the President is authorized to provide for
the creation of a committee of the Manipur Legislative Assembly consisting of the members elected from the
Hill Areas of the state. The President can direct that the Governor shall have special responsibility to secure
the proper functioning of that committee. The President may, by order made with respect to the State of
Manipur, provide for the constitution and functions of a committee of the Legislative Assembly of the State
consisting of members of that Assembly elected from the Hill Areas of that State, for the modifications to be
made in the rules of business of the Government and in the rules of procedure of the Legislative Assembly of
the State and for any special responsibility of the Governor in order to secure the proper functioning of such
committee.
● Statement 2 is incorrect: Under article 371-C the Governor has responsibility to submit an annual report to
the President regarding the administration of the Hill Areas of Manipur. The Central Government can give
directions to the State Government as to the administration of the Hill Areas.

Copy is ENCRYPTED and for personal use ONLY| www.sunyaias.com |MANTHAN-2024 Page. 9
MANTHAN-2024 | PRELIMS TEST SERIES | pPLT-3.0
22. Answer: (B) Only two pairs are correct.
Explanation:
● To fulfill the vision of Article 40 i.e. ‘organization of village panchayats’; the government took several steps
including constituting various committees to give panchayats a structure. This ultimately culminated in the
73rd and 74th Constitutional Amendment, establishing Panchayati Raj Institutions (PRIs) in India.
● Pair 1 is incorrect: In January 1957, the Government of India appointed a committee to examine the working
of the Community Development Programme (1952) and the National Extension Service (1953) and to suggest
measures for their better working. The chairman of this committee was Balwant Rai G Mehta. It recommended
establishment of a three-tier panchayati raj system–gram panchayat at the village level, panchayat samiti at
the block level and zila parishad at the district level. Mandal Panchayat was recommended by Ashok Mehta
committee.
● Pair 2 is incorrect: In December 1977, the Government appointed a committee on Panchayat Raj Institutions
under the chairmanship of Ashok Mehta. Its key recommendation was that the three-tier system of Panchayat
raj should be replaced by the two-tier system, that is, zila Parishad at the district level, and below it, the mandal
panchayat consisting of a group of villages with a total population of 15,000 to 20,000.
● Pair 3 is correct: The Committee to review the existing Administrative Arrangements for Rural Development
and Poverty Alleviation Programmes under the chairmanship of G.V.K. Rao was appointed by the Planning
Commission in 1985. One of its key recommendations was the creation of the post of District Development
Commissioner. And he should act as the chief executive officer of the Zila Parishad and should be in charge
of all the development departments at the district level.
● Pair 4 is correct: In 1986, Rajiv Gandhi government appointed a committee to prepare a concept paper on
‘Revitalisation of Panchayati Raj Institutions for Democracy and Development’ under the chairmanship of
L.M. Singhvi. It recommended that the Panchayati Raj institutions should be constitutionally recognised,
protected and preserved. For this purpose, a new chapter should be added in the Constitution of India. This
will make their identity and integrity reasonably and substantially inviolate. It also suggested constitutional
provisions to ensure regular, free and fair elections to the Panchayati Raj bodies.

23. Answer: (A) Only one statement is incorrect.


Explanation:
● Statement 1 is correct: The formation of District Planning Committees (DPCs) is mandated by the Article
243ZD of the 74th Amendment of the Constitution. District Planning Committees serve as a link between
Panchayats and Urban Local Bodies. District Planning Committees (DPCs) consolidate the plans prepared by
panchayats and municipalities in the district, and to prepare a draft development plan for the district as a
whole. Every District Planning Committee in preparing the draft development plan, deals with matters of
common interest between the Panchayats and the Municipalities including spatial planning, sharing of water
and other physical and natural resources, the integrated development of infrastructure and environmental
conservation. It also discusses the extent and type of available resources whether financial or otherwise.
● Statement 2 is correct: It is the State legislature that may make provisions with respect to the composition
of district planning committees. Furthermore, state legislature can also make provisions with respect to the
manner of election of members of such committees, the functions of such committees in relation to district
planning and the manner of the election of the chairpersons of such committees.
● Statement 3 is incorrect: The four-fifths of the members of a district planning committee are elected by the
elected members of the District Panchayat and municipalities in the district from amongst themselves.
(Through this we can say that, the governor does not nominate 30% of the members to DPC). The
representation of these members in the committee is in proportion to the ratio between the rural and urban
populations in the district.

24. Answer: (C) Only three statements are correct.


Explanation:
● Option 1 is correct: Property tax is the amount that is paid by the landowner to the municipal corporation or
the local government for his/her area. The tax must be paid every year. Property, office buildings, and
residential homes that are rented out to third parties are considered real estate assets. Property tax is charged
Copy is ENCRYPTED and for personal use ONLY| www.sunyaias.com |MANTHAN-2024 Page. 10
MANTHAN-2024 | PRELIMS TEST SERIES | pPLT-3.0
by the government on all tangible real estate that an individual owns. These real estate assets could include
residential homes, office buildings and premises rented out to third parties. It also known as house tax.
● Option 2 is correct: The non-tax revenue for local government comes from municipal properties, fees and
fines, royalty, profits and dividends, interest, user charges and miscellaneous receipts. The user charges (i.e.,
payment for public utilities) include water charges, sanitation charges, and sewerage charges and so on.
● Option 3 is correct: The urban local bodies raise loans from the state government as well as financial
institutions to meet their capital expenditure. They can borrow from the financial institutions or other bodies
only with the approval of the state government.
● Option 4 is incorrect: Excise duty was a form of indirect tax that was levied by the Central Government of
India for the production, sale, or license of certain goods prior to GST. However, Excise duty charges continue
to be in force for alcohol and narcotics which are collected by state governments and not local government.

25. Answer: (A) Only one statement is correct.


Explanation:
● Statement 1 is correct: The Panchayat raj system (system of local governance) was given constitutional
sanction, through the 73rd and 74th Constitutional Amendment Acts of 1992. While the constitution makes
some provisions mandatory for all panchayats in all states, some provisions are voluntary and have been left
to the discretion of the state legislatures to decide as they see fit, for panchayats in their area. One such matter
is the matter of Finances of a Panchayat. The constitution does not confer any particular tax raising powers
on the Panchayats. It is for the state legislatures to decide which taxes the panchayats in that state are eligible
to levy and collect.
● Statement 2 is incorrect: India is a federal polity in its working. This means there is a distribution of power
among various levels of governance - like national, state, and local. So, to clarify the roles, responsibilities,
authorities and limits of various levels of government, our constitution divided various subjects which could
be legislated upon into 3 lists - Union, State & Concurrent, placed in the 7th Schedule to the constitution. The
73rd and 74th constitutional amendment acts of 1992, placed local governance in the State (not the
Concurrent) List. Thus, matters related to local governance (both rural and urban) are placed under the
legislative competence of the state (i.e., that the State Legislature makes laws on matters deciding about
various issues related to local government). Hence this statement is incorrect.
● Statement 3 is incorrect: The act provides for a Gram Sabha as the foundation of the Panchayat raj system.
It is a body consisting of persons registered in the electoral rolls of a village comprised within the area of
Panchayat at the village level. Thus, it is a village assembly consisting of all the registered voters in the area
of a panchayat. It may exercise such powers and perform such functions at the village level as the legislature
of a state determines. Empowering the Gram Sabha could have been a powerful weapon for transparency,
accountability and for involvement of the marginalized sections. However, a number of the State Acts have
not spelt the powers of Gram Sabha nor have any procedures been laid down for the functioning of these
bodies or penalties for the officials.

26. Answer: (B) Only two Statements are correct.


Explanation:
● Statement 1 and 2 are correct: The 73rd Amendment Act was passed in 1992 and came into effect on 24th
April 1993 which inserted Part IX consisting of Article 243 to 243-O, and the Eleventh Schedule enumerating
29 functional items. The Act gave certain powers to the state government to constitute gram panchayats at a
local level and provide them with all the necessary assistance to operate as a unit of self-governance.
● The compulsory (obligatory or mandatory) provisions of the 73rd Constitutional Amendment Act (1992) or
the Part IX of the Constitution are:
I. Organization of Gram Sabha in a village or group of villages.

Copy is ENCRYPTED and for personal use ONLY| www.sunyaias.com |MANTHAN-2024 Page. 11
MANTHAN-2024 | PRELIMS TEST SERIES | pPLT-3.0
II. Establishment of panchayats at the village, intermediate and district levels.
III. Direct elections to all seats in panchayats at the village, intermediate and district levels.
IV. Indirect elections to the post of chairperson of panchayats at the intermediate and district levels.
V. 21 years to be the minimum age for contesting elections to panchayats.
VI. Reservation of seats (both members and chairpersons) for SCs and STs in panchayats at all the three
levels.
VII. Reservation of one-third seats (both members and chairpersons) for women in panchayats at all the
three levels.
VIII. Fixing tenure of five years for Panchayat at all levels and holding fresh elections within six months in
the event of supersession of any Panchayat.
IX. Establishment of a State Election Commission for conducting elections to the Panchayat.
X. Constitution of a State Finance Commission after every five years to review the financial position of
the Panchayat.
● Statement 3 is incorrect: Giving representation to members of the Parliament (both the Houses) and the state
legislature (both the Houses) in the panchayats at different levels falling within their constituencies are the
voluntary provisions under 73rd Constitutional Amendment Act.

27. Answer: A
Explanation:
● Option A is correct: The Fifth Schedule of the Constitution deals with the administration and control of
Scheduled Areas as well as of Scheduled Tribes residing in any State other than the States of Assam,
Meghalaya, Tripura and Mizoram. Governor can make regulations for the peace and good government of a
scheduled area after consulting the tribe's advisory council. Such regulations may prohibit or restrict the
transfer of land by tribal to non-tribal members or among members of the scheduled tribes, regulate the
allotment of land to members of the scheduled tribes.
● Option b is incorrect: According to the provisions of Paragraph 4, under Article 244(1) of Fifth Schedule of
the Constitution of India, the Tribes Advisory Councils (TAC) shall be established in each State having
Scheduled Areas therein and, if the President so directs, also in any State having Scheduled Tribes but not
Scheduled Areas. Tribal advisory council is an advisory body, not a governing body.
● Option c is incorrect: Bringing any particular area under the Fifth Schedule of the Constitution of India does
not convert the area into a Union Territory.
● Option d is incorrect: Bringing any particular area under the Fifth Schedule of the Constitution of India does
not get declared as a Special Category State.

28. Answer: (B) Only two Statements are correct.


Explanation:
● Statement 1 is incorrect: According to Article 300 of the Indian Constitution, an individual or organization
can file a lawsuit in the High Court or Supreme Court against the central government by the name of the
Union of India not by the Government of India, and in case of the Government of a State, it's by the name of
the State by sending 2 months’ prior notice under Section 80 of the CPC.
● Statement 2 is the correct: As per the exercise of its executive power, the Union or a state can enter into
contracts for the acquisition, holding and disposal of property, or to carry on any trade or business, or for any
other purpose with any organization. But any breach of the contract does not immunize the government from
a contractual liability, making the government suable in contracts. This means that the contractual liability of
the Union government and the state governments is the same as that of an individual under the ordinary law
of contract. Hence the government in India can be sued by the organization on breach of contract from their
part.
● Statement 3 is the correct: As per the ruling in the P and O Steam Navigation Company case (1861) it was
stated that the government (Union or states) in India can be sued for civil wrongs done by the government
Copy is ENCRYPTED and for personal use ONLY| www.sunyaias.com |MANTHAN-2024 Page. 12
MANTHAN-2024 | PRELIMS TEST SERIES | pPLT-3.0
officials only in the exercise of its non-sovereign functions but not in the sovereign functions like
administering justice, constructing a military road. Commandeering goods during war, etc. It was reaffirmed
by the Supreme Court in the post-independence era in the Kasturi Lal case (1965).

29. Answer: (B) Only two Statements are correct.


Explanation:
● Statement 1 is incorrect: Citizen Charters is voluntary and written document that spells out the service
provider’s efforts taken to focus on their commitment towards fulfilling the needs of the citizens/customers.
A Model Panchayat Citizens Charter/ framework for delivery of the services across the 29 sectors of 11th
Schedule of Constitution, aligning actions with localised Sustainable Development Goals (SDGs). Model
Panchayat Citizens Charter framework has been prepared by the Ministry of Panchayati Raj (MoPR) (not the
Ministry of Rural Development) in collaboration with National Institute of Rural Development &Panchayati
Raj (NIRDPR). NIRDPR is an autonomous organization under the Union Ministry of Rural Development.
● Statement 2 is correct: Model Panchayat Citizens Charter framework has been developed for the delivery of
the services across the 29 sectors (mentioned under schedule 11 of the Constitution for Panchayats), and it
aims to align the delivery of these public services with Sustainable Development Goals at panchayat level.
● Statement 3 is correct: Panchayats would be utilizing this framework, and with the due approval of Gram
Sabha, would draw up a Citizens Charter, detailing the different categories of services rendered to the citizen
by the Panchayat, the conditions for such service and also the time limit for such service.

30. Answer: (A) Only one statement is correct.


Explanation:
● Statement 1 is incorrect: Each autonomous district has a district council consisting of 30 members, of whom
four are nominated by the Governor and the remaining 26 are elected on the basis of adult franchise. The
elected members hold office for a term of five years and nominated members hold office during the pleasure
of the Governor. Each autonomous region also has a separate regional council.
● Statement 2 is correct: The district and regional councils administer the areas under their jurisdiction. They
can make laws on certain specified matters like land, forests, canal water, shifting cultivation, village
administration, inheritance of property, marriage and divorce, social customs and so on. The district council
can establish, construct or manage primary schools, dispensaries, markets, ferries, fisheries, roads and so on
in the district. It can also make regulations for the control of money lending and trading by non-tribals. But
all such laws require the assent of the Governor.
● Statement 3 is incorrect: The Governor can appoint a commission to examine and report on any matter
relating to the administration of the autonomous districts or regions. Governor may dissolve a district or
regional council on the recommendation of the commission.

31. Answer: (B) Only two Statements are correct.


Explanation:
● Statement 1 is incorrect: The Scheduled Areas are defined in the Fifth Schedule are inhabited by
‘aboriginals’ who are socially and economically rather backward and special efforts need to be made to
improve their condition. The President is empowered to declare an area to be a scheduled area. He can also
increase or decrease its area, alter its boundary lines, rescind such designation or make fresh orders for such
re-designation on an area in consultation with the Governor of the state concerned.
● Statement 2 is incorrect: The executive power of a state extends to the Scheduled Areas. But the Governor
has a special responsibility regarding such areas. He has to submit a report to the President regarding the
administration of such areas, annually or whenever so required by the President. The executive power of the
Centre extends to giving directions to the states regarding the administration of such areas.
● Statement 3 is correct: Each state having Scheduled Areas has to establish a tribe's advisory council to advice
on welfare and advancement of the scheduled tribes. A similar council can also be established in a state having

Copy is ENCRYPTED and for personal use ONLY| www.sunyaias.com |MANTHAN-2024 Page. 13
MANTHAN-2024 | PRELIMS TEST SERIES | pPLT-3.0
scheduled tribes but not Scheduled Areas therein, if the President so directs. It is to consist of 20 members,
three-fourths of whom are to be the representatives of the scheduled tribes in the state legislative assembly.
● Statement 4 is correct: The Governor have several powers in respect to the Scheduled Areas. He may make
a regulation to repeal or amend any act of Parliament or the state legislature, which is applicable to a scheduled
area. But all such regulations require the assent of the President.

32. Answer: (B) Only three Statements are correct.


Explanation:
● Statement 1 and 3 is correct: Article 330 and 332 of the Indian constitutions provides for the Reservation
of seats for scheduled castes and scheduled tribes in the House of the people (Lok Sabha) and legislative
assemblies of the states respectively. The Constitution provided for the reservation of seats for scheduled
castes and scheduled tribes in the house of people and assembly of each state on the basis of population ratios.
Parliament in 2019 passed a Constitutional amendment bill to extend quota to SCs and STs in Lok Sabha and
state assemblies by another 10 years. Reservation for members of Scheduled Castes (SCs) and Scheduled
Tribes (STs), given for the past 70 years in Lok Sabha and state assemblies was due to end on January 25,
2020.
● Statement 2 is incorrect: The Indian constitution does not prescribe the reservation of seats for scheduled
castes and scheduled tribes in Rajya Sabha.
● Statement 4 is correct: 73rd constitutional amendment act of 1992 has provided for the reservation of seats
for scheduled castes and scheduled tribes in every panchayat (I.e., at all the three levels) in proportion of their
population to the total population in the panchayat area. The state legislature shall provide for the reservation
of offices of chairperson in the panchayat at the village or any other level for the SCs and STs.
● Statement 5 is incorrect: The 74th constitutional amendment act provides for the reservation of seats for the
scheduled castes and the scheduled tribes in every municipality in proportion of their population to the total
population in the municipal area. The offices of Chairpersons in the Municipalities shall be reserved for the
Scheduled Castes, the Scheduled Tribes and women in such manner as the Legislature of a State may, by law,
provide. Hence reservation for chairpersons of urban local bodies is based on the discretion of State
Legislatures and not mandatory as per the constitution.
33. Answer: (A) Only one statement is correct.
Explanation:
● Statement 1 is incorrect: Article 339 of Indian constitution extends the Control of the Union over the
administration of Scheduled Areas and the welfare of Scheduled Tribes. The President (not the Parliament)
may at any time appoint a Commission to report on the administration of the Scheduled Areas and the welfare
of the Scheduled Tribes in the States. He can appoint such a commission at any time but compulsorily after
ten years of the commencement of the Constitution. The order may define the composition, powers and
procedure of the Commission and may contain such incidental or ancillary provisions as the President may
consider necessary or desirable.
● Statement 2 is correct: The executive power of the Union shall extend to the giving of directions to a State
as to the drawing up and execution of schemes specified in the direction to be essential for the welfare of the
Scheduled Tribes in the State.
● Statement 3 is incorrect: Two commissions were appointed by the President regarding the administration of
Scheduled Areas and the welfare of Scheduled Tribes. First commission was appointed in 1960. It was headed
by U.N. Dhebar (not DB Kalelkar) and submitted its report in 1961.The second commission was appointed in
2002 under the chairmanship of Dilip Singh Bhuria. It submitted its report in 2004.

34. Answer: C
Explanation:
● Option C is correct: The Constitution, under Sixth Schedule, contains special provisions for the
administration of tribal areas in the four northeastern states of Assam, Meghalaya, Tripura and Mizoram.

Copy is ENCRYPTED and for personal use ONLY| www.sunyaias.com |MANTHAN-2024 Page. 14
MANTHAN-2024 | PRELIMS TEST SERIES | pPLT-3.0
The Tribal Areas under Sixth Schedule include:
⮚ Assam - North Cachar Hills District, KarbiAnglong District and Bodoland Territorial Areas District.
⮚ Meghalaya - Khasi Hills District, Jaintia Hills District and Garo Hills District.
⮚ Tripura - Tripura Tribal Areas District.
⮚ Mizoram - Chakma District, Mara District and Lai District.
● In Assam, the Statutory Autonomous Councils are constituted for Social, Economic, Educational, Ethnic and
Cultural advancement of the Scheduled Tribe (ST) communities living in Core Areas as well as in Satellite
Areas covering many districts of Assam. There are six Statutory Autonomous Councils:
I. RabhaHasong Autonomous Council
II. Mising Autonomous Council
III. Tiwa Autonomous Council
IV. Deori Autonomous Council
V. ThengalKachari Autonomous Council
VI. SonowalKachari Autonomous Council.
The tribes in Assam, Meghalaya, Tripura and Mizoram, still have their roots in their own culture, customs and
civilization. These areas are, therefore, treated differently by the Constitution and sizeable amount of autonomy
has been given to these people for self-government.
35. Answer: (B) Only two Statements are incorrect.
Explanation:
● Statement 1 is incorrect: Any citizen can file a public interest litigation by filing a petition: Under Art 32 of
the Indian Constitution, in the Supreme Court Under Art 226 of the Indian Constitution, in the High Court
Under 133 of the Criminal Procedure Code, in a Magistrate ‘s Court
● Statement 2 is incorrect: A PIL may be filed against state government, central government, municipal
authority, private party. Also, private person may be included in PIL as ‗Respondent ‘, after concerned of
state authority. Foreg. Aprivate factory in Mumbai which is causing pollution then PIL can be filed against
the government of Mumbai, state pollution central board including that private factory of Mumbai.
● Statement 3 is correct: PIL is not defined in any statute or in any act. PIL gives a wider description to the
fundamental rights to equality, life and personality, which is guaranteed under part III of the Constitution of
India. It also functions as an effective instrument for changes in the society or social welfare. Any public or
person can seek remedy on behalf of the oppressed class by introducing a PIL. Before 1980s the judiciary and
the Supreme Court of India entertained litigation only from parties affected directly or indirectly by the
defendant. It heard and decided cases only under its original and appellate jurisdictions. It is the chief
instrument through which judicial activism has flourished in India. It is suited to the principles enshrined in
Article 39A[a] of the Constitution to protect and deliver prompt social justice with the help of law.

36. Answer: (B) Only two Statements are correct.


Explanation:
● Statement 1 is incorrect: As per Article 130, The Supreme Court shall sit in Delhi or in such other place or
places, as the Chief Justice of India may, with the approval of the President, from time to time.
● Statement 2 is correct: Being a Court of Record, the High Court can review its own judgments under Article
226 of the Constitution of India. Similarly, under Article 137, the Supreme Court shall have the power to
review any judgment pronounced or order made by it.
● Statement 3 is correct: As per Article128 of Indian Constitution, the Chief Justice of India may at any time,
with the previous consent of the President, request any person to sit and act as a Judge of the Supreme Court
with the following qualifications:
⮚ Who has held the office of a Judge of the Supreme Court.
⮚ Who has held the office of a Judge of a High Court and is duly qualified for appointment as a Judge of
the Supreme Court.

Copy is ENCRYPTED and for personal use ONLY| www.sunyaias.com |MANTHAN-2024 Page. 15
MANTHAN-2024 | PRELIMS TEST SERIES | pPLT-3.0
● Chief Minister of Tamil Nadu, M.K. Stalin recently reiterated the State’s request for establishing a Regional
Bench of the Supreme Court in Chennai and allowing Tamil to be used in the Madras High Court as one of
its official languages.
Constitutional Provisions for Judicial Review
⮚ There is no direct and express provision in the constitution empowering the courts to invalidate laws, but
the constitution has imposed definite limitations upon each of the organs, the transgression of which
would make the law void.
⮚ The court is entrusted with the task of deciding whether any of the constitutional limitations has been
transgressed or not. Some provisions in the constitution supporting the process of judicial review are:
⮚ Article 372 (1) establishes the judicial review of the pre constitution legislation.
⮚ Article 13 declares that any law which contravenes any of the provisions of the part of Fundamental
Rights shall be void.
37. Answer: (B) Only two Statements are correct.
Explanation:
● Statement 1 is incorrect: Constitution bench is the name given to the benches of the Supreme Court of India
which consist of at least five judges of the court which sit to decide any case “involving a substantial question
of law as to the interpretation” of the Constitution of India or "for the purpose of hearing any reference" made
by the President of India under Article 143.
● Statement 2 is correct: The Constitution Benches are set up on an ad hoc basis and when the need arises.
Constitution Bench is constituted in rare cases to decide important questions of fact or legal and/or
constitutional interpretation.
● Statement 3 is correct: A bench of two judges is called a division bench and a bench of three or five judges
is called full bench. Recently, Justice Uday Umesh Lalit, 49th Chief Justice of India (CJI), assured there will
be at least one Constitution Bench functioning throughout the year in the Supreme Court.
Circumstances for Constitutional Bench to Exist:
● Article 145(3) provides, ―The minimum number of Judges who are to sit for the purpose of deciding any
case involving a substantial question of law as to the interpretation of this Constitution or for the purpose of
hearing any reference under Article 143 shall be five.

38. Answer: D
Explanation:
● Statement 1 is incorrect: As per the Civil Procedure Code and the Supreme Court Rules, any person
aggrieved by a ruling can seek a review. This implies that it is not necessary that only parties to a case can
seek a review of the judgment.
A Review Petition has to be filed within 30 days of the date of judgment or order.
In certain circumstances, the court can condone the delay in filing the review petition if the petitioner can establish
strong reasons that justify the delay.
● (Note: It needs to be noted that judgment is a final decision in a case whereas order is an interim ruling that
is subject to its final judgment)
● Grounds for Considering Review Petition
● It needs to be noted that the Court does not entertain every review petition filed. It exercises its discretion to
allow a review petition only when it shows the grounds for seeking the review.
The Supreme Court has laid down three grounds for seeking a review of a verdict it has delivered:
✔ The discovery of new and important matter or evidence which, after the exercise of due diligence, was not
within the knowledge of the petitioner or could not be produced by him;
✔ Mistake or error apparent on the face of the record; or
✔ Any other sufficient reason that is analogous to the other two grounds.

Copy is ENCRYPTED and for personal use ONLY| www.sunyaias.com |MANTHAN-2024 Page. 16
MANTHAN-2024 | PRELIMS TEST SERIES | pPLT-3.0
● Procedure in the Court
⮚ Review petitions are ordinarily being entertained without oral arguments by lawyers. Thus, it is heard
“through circulation” by the judges in their chambers.
⮚ However, in exceptional cases, the court allows an oral hearing. In a 2014 case, the Supreme Court held
that review petitions in all death penalty cases will be heard in open court by a Bench of three judges.
⮚ Review petitions are also heard by the same combination of judges who delivered the original order or
judgment that is sought to be reviewed.
● Statement 2 is incorrect: The court has the power to review its rulings to correct a "patent error" and not
"minor mistakes of inconsequential import". That means the Court is allowed not to take fresh stock of the
case but to correct grave errors that have resulted in the miscarriage of justice.

39. Answer: (A) Only one statement is incorrect.


Explanation:
● Statement 1 is correct: The Benefits of Judicial Activism Through the Public Interest Litigation (PIL), the
court has expanded the idea of rights. For example, Clean air, unpolluted water, decent living, etc., are the
rights for the entire society. Following its constitutional obligation, the Indian judiciary has actively defended
individuals’ fundamental rights whenever necessary from the state’s unjust, unreasonable, and unfair actions
or inactions through this powerful tool.
● Statement 2 is correct: Through PIL and judicial activism of the post-1980 period, the judiciary has also
shown readiness to take into consideration the rights of those sections who cannot easily approach the courts.
It has democratized the judicial system by giving not just to the individuals, but also the groups access to the
courts.
● Statement 3 is incorrect: Judicial activism has blurred the line of distinction between the executive and the
legislature on one hand and the judiciary on the other. The court has been involved in resolving questions
which belong to the executive. It has overburdened the courts.
The Disadvantage of Judicial Activism
⮚ Violation of Separation of Powers: Judicial Activism violates the principle of separation of powers as
it may result in the judiciary encroaching upon the functions of the executive and legislative branches of
the government. The judiciary is not elected and should not take decisions that should be left to elected
representatives.
⮚ Judicial Overreach: It may result in judicial overreach, where the courts take decisions that are beyond
their competence or legal expertise.
⮚ Delayed Justice: Judicial activism may lead to delays in the administration of justice, as the courts may
take time to deliberate and reach decisions. This can result in justice being denied to those who are in
urgent need of it.
⮚ Lack of Accountability: It reduces the accountability of the judiciary as there is no oversight mechanism
to check the decisions of the judiciary.
Difference between Judicial Review and Judicial Activism:
Judicial review and Judicial Activism are two related concepts in the field of law and governance. While they both
refer to the role of courts in interpreting and applying the law, they have different implications and outcomes.
⮚ Judicial Review: If a legislation, policy, or action taken by the government or another public authority is
deemed to be unlawful, the judiciary has the authority to evaluate it and invalidate it. This power is based
on the doctrine of separation of powers, which holds that each branch of government must act within its
own constitutional limits.
⮚ Judicial Activism: It refers to the role of the judiciary in interpreting and enforcing the law in a way that
promotes social justice and protects fundamental rights. Judicial activism involves judges taking an active
role in the legal and policy-making process, rather than just interpreting and applying existing laws.
Copy is ENCRYPTED and for personal use ONLY| www.sunyaias.com |MANTHAN-2024 Page. 17
MANTHAN-2024 | PRELIMS TEST SERIES | pPLT-3.0
40. Answer: (B) Only two pairs are correct.
Explanation:
● Pair 1 is incorrect: The Doctrine of Harmonious Construction is judicial approach interpretation of differing
conflicting statutes or laws. If two or more provisions of the same statute are repugnant to each other, then in
such a situation the court, if possible, will try to construe the provisions in such a manner as to give effect to
both the provisions by maintaining harmony between the two. The question that the two provisions of the
same statute are overlapping or mutually exclusive may be difficult to determine.
● Pair 2 is correct: The Doctrine of Pith and Substance: The basic purpose of this Doctrine is to determine
under which head of power or field, i.e., under which List (given in the Seventh Schedule) a given piece of
legislation falls. Pith means 'true nature' or 'essence of something' and Substance means 'the most important
or essential part of something'. The Doctrine of Pith and Substance says that where the question arises of
determining whether a particular law relates to a particular subject (mentioned in one List or another), the
court looks to the substance of the matter. Thus, if the substance falls within Union List, then the incidental
encroachment by the law on the State List does not make it invalid.
● Pair 3 is correct: The Doctrine of Colourable Legislation tests the competence of the legislature against an
enacted law. This Doctrine is based upon the Latin maxim "Quandoaliquidprohibetur ex directo, prohibetur
et per obliquum", which means "you cannot do indirectly what you cannot do directly", and based on the
Doctrine of Separation of Powers.
● Doctrine of the sovereignty of the Parliament: The British system is based on the doctrine of the sovereignty
of Parliament, while the Parliament is not supreme in India and enjoys limited and restricted powers due to a
written Constitution, federal system, judicial review and fundamental rights
● Doctrine of Legal Precedent: Doctrine of Legal precedent declares that cases must be decided the same way
when their material facts are the same.
● Doctrine of Eclipse: The doctrine of Eclipse is applied in relation to a pre constitutional law which was valid
when it was enacted. Subsequently when the Constitution came into force a shadow falls on it because it is
inconsistent with the Constitution. The act is eclipsed.
● Doctrine of severability: Under the doctrine of severability, only the offending provision of the act would be
declared void and not the whole act. In other words, there is no need to nullify the whole act if only a part of
it violates fundamental rights. Only the offending part needs to be nullified.

41. Answer: (B) Only three Statements are correct.


Explanation:
Essential aspects of natural justice:
● Statement 1 is correct: Right to fair hearing. Audi Alteram Partem (Rule of Fair Hearing) In simple words,
this rule states that both parties must have the chance to represent their viewpoints, and authorities should
conduct a fair trial accordingly.
● Statement 2 is correct: Recusal: It is the act of abstaining from participation in an official action such as a
legal proceeding due to a conflict of interest of the presiding court official or administrative officer. (Note:
Judges recusing themselves in case of vested interests is a Natural Justice.)
Reason for Recusal:
⮚ When there is a conflict of interest, a judge can withdraw from hearing a case to prevent creating a
perception that he carried a bias while deciding the case.
⮚ The conflict of interest can be in many ways such as:
⮚ Having a prior or personal association with a party involved in the case.
⮚ Appeared for one of the parties involved in a case.
⮚ Ex parte communications with lawyers or non-lawyers.
⮚ An appeal is filed in the SC against a judgment of a High Court (HC) that may have been delivered by
the SC judge when he was in the HC.
Copy is ENCRYPTED and for personal use ONLY| www.sunyaias.com |MANTHAN-2024 Page. 18
MANTHAN-2024 | PRELIMS TEST SERIES | pPLT-3.0
⮚ In a matter of a company in which he holds shares unless he has disclosed his interest and there is no
objection to it.
⮚ The practice stems from the cardinal principle of due process of law that nobody can be a judge in her
own case.
⮚ Any interest or conflict of interest would be a ground to withdraw from a case since a judge has a duty
to act fair.
● Statement 3 is correct: Judgments given by the courts should be well-reasoned. They ensure that the
decision-making process, which affects any individual, adheres to certain minimum requirements and is fair.
Presenting one's case must be given to an individual before condemning him to undesirable consequences.
(Reasoned decisions must be given by any authority while proceeding against anyone. Reasoned decisions
ensure that the decisions are not arbitrary and also facilitate judicial review of the decisions.
● Statement 4 and 5 are incorrect: Judicial review of the laws and access to cheap justice are not the part of
natural justice.

42. Answer: (A) Only one statement is correct.


Explanation:
● Statement 1 is incorrect: Tribunals are institutions established for discharging judicial or quasi-judicial
duties. The objective may be to reduce the caseload of the judiciary or to bring in subject expertise for
technical matters. The original constitution did not contain provisions with respect to tribunals. The 42nd
Amendment Act of 1976 added a new Part XIV-A to the Constitution. This part is entitled 'Tribunals' and
consists of only two Articles- Article 323 A dealing with administrative tribunals and Article 323B dealing
with tribunals for other matters. The word tribunal is mentioned in the Constitution.
● Statement 2 is incorrect: Article 323A vs 323B. Tribunals under Article 323A can only be established by
parliament whereas Tribunals under 323B can be established both by Parliament and State Legislature.
● Statement 3 is correct: Under Article 323 B a hierarchy of tribunals may be created. But a tribunal under
323A is exclusive to the centre or a State or a set of States. Hence no hierarchy can be created.
● Difference between 323-A and 323-B
⮚ Article 323-A deals with Administrative Tribunals.
⮚ Article 323-B deals with tribunals for other matters.
▪ Under Article 323 B, the Parliament and the state legislatures are authorised to provide for the establishment
of tribunals for the adjudication of disputes relating to the following matters:
● Taxation
● Foreign exchange, import and export
● Industrial and labour
● Land reforms
● Ceiling on urban property
● Elections to Parliament and state legislatures
● Food stuff
● Rent and tenancy rights
Articles 323 A and 323 B differ in the following three aspects:
✔ While Article 323 A contemplates the establishment of tribunals for public service matters only, Article
323 B contemplates the establishment of tribunals for certain other matters (mentioned above).
✔ While tribunals under Article 323 A can be established only by Parliament, tribunals under Article 323
B can be established both by Parliament and state legislatures with respect to matters falling within their
legislative competence.
✔ Under Article 323 A, only one tribunal for the Centre and one for each state or two or more states may
be established. There is no question of the hierarchy of tribunals, whereas under Article 323 B a hierarchy
of tribunals may be created.

Copy is ENCRYPTED and for personal use ONLY| www.sunyaias.com |MANTHAN-2024 Page. 19
MANTHAN-2024 | PRELIMS TEST SERIES | pPLT-3.0
43. Answer: (C) Only three statements are incorrect.
Explanation:
● Statement 1 is correct: Tribunals are quasi-judicial bodies empowered with judicial powers that deals with
different disputes. The provision for tribunals was not present in the Constitution originally. Article 323A
authorizes the Parliament to provide for tribunals. Administrative Tribunals was set-up by an act
Parliament, Administrative Tribunals Act, 1985. It owes its origin to Article 323 A of the Constitution. It
adjudicates disputes and complaints with respect to recruitment and conditions of service of persons appointed
to the public service and posts in connection with the affairs of the Union and the States.
The Administrative Tribunals Act, 1985 provides for three types of tribunals:
✔ The Central Government establishes an administrative tribunal called the Central Administrative Tribunal
(CAT).
✔ The Central Government may, upon receipt of a request in this behalf from any State Government, establish
an administrative tribunal for such State employees.
✔ Two or more States might ask for a joint tribunal, which is called the Joint Administrative Tribunal
(JAT), which exercises powers of the administrative tribunals for such States.
There are tribunals for settling various administrative and tax-related disputes, including:
⮚ Central Administrative Tribunal (CAT), Income Tax Appellate Tribunal (ITAT), Customs, Excise and
Service Tax Appellate Tribunal (CESTAT), National Green Tribunal (NGT), Competition Appellate Tribunal
(COMPAT) and Securities Appellate Tribunal (SAT), among others.
● Statement 2 is incorrect: CAT is a multi-member structure. It consists of a chairperson and members. There
are 19 Benches and 19 Circuit Benches in the Central Administrative Tribunal all over India. The Government
of India has notified 215 organizations including Ministries and Departments of Central Government, under
section 14 (2) of the Administrative Tribunals Act, 1985 to bring them within the jurisdiction of the Central
Administrative Tribunal, from time to time. In addition, the Central Administrative Tribunal, Principal Bench
is dealing with the matters of Govt. of National Capital Territory of Delhi. The Central Administrative
Tribunal is headed by Hon'ble Chairman Sh. Justice Ranjit Vasantrao More, retired Chief Justice, Meghalaya
High Court. There are 69 Hon'ble Members in various Benches of the Tribunal out of which 34 are Judicial
Members and 35 are Administrative Members. Subject to other provisions of the Act, a Bench consists of one
Judicial Member and one Administrative Member. There will be a Chairman, Vice-Chairman and 25 members
in the Central Administrative Tribunal along its branches The Central Administrative Tribunal has been
established as a specialist body comprising of Administrative Members and Judicial Members who by virtue
of their specialized knowledge are better equipped to dispense speedy and effective justice.
● Statement 3 is incorrect: The chairman of both Central, as well as State Tribunals, is appointed by the
President. However, only the Chief Justice of India is consulted for the appointment of Chairman of Central
Administrative Tribunal.
● Statement 4 is incorrect: Members of State tribunals are appointed by the President in consultation with the
Governor of the concerned state.

Copy is ENCRYPTED and for personal use ONLY| www.sunyaias.com |MANTHAN-2024 Page. 20
MANTHAN-2024 | PRELIMS TEST SERIES | pPLT-3.0

44. Answer: A
Explanation:
● Statement 1 is correct: Anticipatory bail can be obtained by a person who anticipates arrest. Hence,
anticipatory bail is a direction to release a person on bail, even before the person is arrested.
● Statement 2 is incorrect: It is issued only by the Sessions Court and High Court.
The concept of anticipatory bail
● The Supreme Court has resolved a dichotomy in approach of high courts and ruled that Courts in
""extraordinary circumstances"" have the discretion to grant protection from arrest to accused even while
denying them anticipatory bail, but the power cannot be exercised in an untrammeled manner, and the order
will have to be a reasoned one.
The Supreme Court ruling:
⮚ HCs and SC are given powers to grant anticipatory bail to the accused because of the premium that the
Constitution places on the right to liberty guaranteed under Article 21.
⮚ The grant or rejection of an application under CrPC has a direct bearing on the right to life and liberty of
an individual. Therefore, the provision needs to be read liberally, and considering its beneficial nature.
The courts must not read in restrictions that the legislature has not explicitly provided for.
⮚ In doing so, the court may also exercise its powers under Article 142 of the Constitution to pass such an
order.
Need for such protection:
● An accused, besides being an accused, may also be the primary caregiver or sole breadwinner of the family.
His arrest may leave his loved ones in a state of starvation and neglect. In the 1980 Gurbaksh Singh Sibbia vs
State of Punjab case, a five-judge Supreme Court bench led by then Chief Justice Y V Chandra chud ruled
that 438 (1) is to be interpreted in the light of Article 21 of the Constitution (protection of life and personal
liberty).

Copy is ENCRYPTED and for personal use ONLY| www.sunyaias.com |MANTHAN-2024 Page. 21
MANTHAN-2024 | PRELIMS TEST SERIES | pPLT-3.0
The concept of anticipatory bail:
● The provision of anticipatory bail under Section 438 was introduced when CrPC was amended in 1973.As
opposed to ordinary bail, which is granted to a person who is under arrest, in anticipatory bail, a person is
directed to be released on bail even before arrest is made.
● Time limit: The Supreme Court (SC) in Sushila Aggarwal v. State of NCT of Delhi (2020) case delivered a
significant verdict, ruling that no time limit can be set while granting anticipatory Bail and it can continue
even until the end of the trial.

45. Answer: D
Explanation:
● Statement 1 is incorrect: The Constitution declares Delhi as the seat of Supreme court. The Constitution
authorises the Chief Justice of India and not the President to appoint places other than Delhi as seat of the
Supreme Court. But yes, he can take decision in this regard only with the approval of President.
● Statement 2 is incorrect: Unless the Parliament provides otherwise, all proceedings of Supreme Court should
be in English. And as the Parliament has not made any provision for the use of Hindi, So Supreme court
allows only those petitions or appeals which are in English only as of now.

46. Answer: B
Explanation:
● Statement 1 is incorrect: Gram Nyayalayas are established under the Gram Nyayalayas Act, 2008 for speedy
and easy access to justice system in the rural areas. The Act came into force from 2 October 2009. They can
try criminal cases as well as civil suits, claims or disputes which are specified in the First Schedule and the
Second Schedule of the Act.
● Statement 2 is correct: The Act explicitly provides for mediation by social workers. The Gram Nyayalaya
shall not be bound by the rules of evidence provided in the Indian Evidence Act, 1872 but shall be guided by
the principles of natural justice and subject to any rule made by the High Court.
● ADDITIONAL INFORMATION:
● Composition:
⮚ The Gram Nyayalayas are presided over by a Nyayadhikari, who will have the same power, enjoy same
salary and benefits of a Judicial Magistrate of First Class.
⮚ Such Nyayadhikari are to be appointed by the State Government in consultation with the respective High
Court.
● Jurisdiction:
⮚ A Gram Nyayalaya have jurisdiction over an area specified by a notification by the State Government in
consultation with the respective High Court.
⮚ The Court can function as a mobile court at any place within the jurisdiction of such Gram Nyayalaya,
after giving wide publicity to that regards.
⮚ They have both civil and criminal jurisdiction over the offences.
⮚ The pecuniary jurisdiction of the Nyayalayas are fixed by the respective High Courts.
⮚ Gram Nyayalayas has been given power to accept certain evidences which would otherwise not be
acceptable under Indian Evidence Act.
● Procedure to be followed:
⮚ Gram Nyayalayas can follow special procedures in civil matters, in a manner it deems just and reasonable
in the interest of justice.
⮚ Gram Nyayalayas allow for conciliation of the dispute and settlement of the same in the first instance.
● Appeals:
⮚ Appeal in criminal cases shall lie to the Court of Session, which shall be heard and disposed of within a period
of six months from the date of filing of such appeal.

Copy is ENCRYPTED and for personal use ONLY| www.sunyaias.com |MANTHAN-2024 Page. 22
MANTHAN-2024 | PRELIMS TEST SERIES | pPLT-3.0
⮚ Appeal in civil cases shall lie to the District Court, which shall be heard and disposed of within a period of
six months from the date of filing of the appeal.
● Significance:
The setting up of Gram Nyayalayas is considered as an important measure to reduce arrears and is a part of the
judicial reforms. It is estimated that Gram Nyayalayas can reduce around 50% of the pendency of cases in
subordinate courts and can take care of the new litigations which will be disposed within six months.

47. Answer: (A) Only one statement is correct.


Explanation:
● Judicial Restraint is a theory of judicial interpretation that encourages judges to limit the exercise of their own
power. It asserts that judges should hesitate to strike down laws unless they are obviously unconstitutional.
Judicially-restrained judges respect stare-decisis, the principle of upholding established precedent handed
down by past judges.
● Statement 1 is incorrect: While delivering a judgment in December 2007, the Supreme Court of India called
for judicial restraint and commented that judicial restraint is consistent with and complementary to the balance
of power among the three independent branches of the state.
● Statement 2 is incorrect: The Supreme Court has stated that judicial restraint tends to protect the
independence of the judiciary. When courts encroach into the legislative or administrative fields almost
inevitably voters, legislators, and other elected officials will conclude that the activities of judges should be
closely monitored. If judges act like legislators or administrators, it follows that judges should be elected like
legislators or selected and trained like administrators. This would be counterproductive. The touchstone of an
independent judiciary has been its removal from the political or administrative process. Even if this removal
has sometimes been less than complete, it is an ideal worthy of support and one that has had valuable effects.
● Statement 3 is correct: The Supreme Court has mentioned that judicial restraint recognizes the equality of
the other two branches (Legislature and Executive) with the judiciary. Judicial restraint also fosters that
equality by minimizing inter-branch interference by the judiciary.
48. Answer: (B) Only two Statements are correct.
Explanation:
● Statement 1 is incorrect: According to provisions of Judges (Inquiry) Act, 1968, the motion to impeach a
judge of the Supreme Court of India can be rejected by the Speaker of the Lok Sabha.
● Statement 2 is incorrect: The Constitution of India does not define or give details of what constitutes
‘incapacity and proved misbehaviour’ of the Judges of the Supreme Court of India.
● Statement 3 is correct: The Judges Enquiry Act (1968) regulates the procedure relating to the removal of a
judge of the Supreme Court by the process of impeachment. It states that a removal motion signed by 100
members (in the case of Lok Sabha) or 50 members (in the case of Rajya Sabha) is to be given to the Speaker/
Chairman.
● Statement 4 is correct: If the motion for the impeachment of a Judge is taken up for voting, the law requires
the motion to be backed by each House of the Parliament and supported by a majority of total membership of
that House and by not less than two-thirds of total members of that House present and voting. After the motion
is passed by each House of Parliament, an address is presented to the president for removal of the judge.
Finally, the president passes an order removing the judge.

49. Answer: D
Explanation:
1. Statement A is incorrect: Lok Adalat is one of the alternative dispute redressal mechanisms, it is a forum
where disputes/cases pending in the court of law or at pre-litigation stage are settled/ compromised amicably.
There is no court fee payable when a matter is filed in a Lok Adalat. If a matter pending in the court of law is
referred to the Lok Adalats and is settled subsequently, the court fee originally paid in the court on the
complaints/petition is also refunded back to the parties.

Copy is ENCRYPTED and for personal use ONLY| www.sunyaias.com |MANTHAN-2024 Page. 23
MANTHAN-2024 | PRELIMS TEST SERIES | pPLT-3.0
2. Statement B is incorrect: Lok Adalats can take cognizance matters that are both civil or criminal in nature,
but any matter relating to an offence not compoundable under any law cannot be decided by the Lok Adalat
even if the parties involved therein agree to settle the same. It not only deals with those persons who are
entitled to avail free legal services but of all other persons also, be they women, men, or children and even
institutions.
3. Statement C is incorrect: Every State Authority or District Authority or Supreme Court Legal Services
Committee or every High Court Legal Services Committee or Taluk Legal Services Committee may organize
Lok Adalats. Lok Adalats are to consist of three members – a sitting or retired judicial officer, a member of
the legal profession (advocate, law officer, and law teacher) and a social worker, preferably women. The Act
and regulations require the secretary of the legal services authority or committee to associate students, social
activists and voluntary organisations in the community for facilitating the successful conduct of ‘Lok Adalats’.
4. Statement D is correct: Permanent Lok Adalats are organized under Section 22-B of The Legal Services
Authorities Act, 1987. Permanent Lok Adalats have been set up as permanent bodies with a Chairman and
two members for providing compulsory pre-litigative mechanism for conciliation and settlement of certain
cases. These cases are relating to Public Utility Services like transport, postal, telegraph etc. Here, even if the
parties fail to reach to a settlement, the Permanent Lok Adalat gets jurisdiction to decide the dispute, provided,
the dispute does not relate to any offence. Further, the Award of the Permanent Lok Adalat is final and binding
on all the parties. The jurisdiction of the Permanent Lok Adalats is upto Rs. 10 Lakhs.
50. Answer: (C) Only four statements are incorrect.
Explanation:
Contempt of court can be of two kinds:
⮚ Civil, that is the willful disobedience of a court order or judgment or willful breach of an undertaking
given to a court.
Criminal Contempt of court means publication of any matter which:
⮚ Scandalises or tends to scandalise, or lowers or tends to lower the authority of, any court.
⮚ Prejudices, or interferes or tends to interfere with the due course of any judicial proceeding.
⮚ Interferes or tends to interfere with, or obstructs or tends to obstruct, the administration of justice in any
other manner.
● Statement 1 is incorrect: Fair and reasonable criticism of judicial acts do not amount to Contempt of court.
● Statement 2 is correct: An act which interferes or prejudices with due course of judicial proceedings is
regarded as Criminal Contempt of court.
● Statement 3 is incorrect: Commenting on administrative side of Judiciary do not amount to Contempt of
court.
● Statement 4 is incorrect: Innocent publication and discussion of matters of court also don’t come in the
category of Contempt of court.
● Statement 5 is incorrect: Disobeying willfully any judgment of court comes in the category of Civil
Contempt (not criminal) of court.
Provisions for Court with respect to enforcement of this concept:
✔ Article 129 and 215 of the Constitution of India empowers the Supreme Court and High Court respectively
to punish people for their respective contempt. Article 142 of the Indian Constitution also empowers the court
to punish for its contempt.
✔ Section 10 of The Contempt of Courts Act of 1971 defines the power of the High Court to punish contempt’s
of its subordinate courts.
✔ The Constitution also includes contempt of court as a reasonable restriction to the freedom of speech and
expression under Article 19, along with elements like public order and defamation.
(NOTE: The Constitution does not provide a definition of “contempt of court.”)

******
Copy is ENCRYPTED and for personal use ONLY| www.sunyaias.com |MANTHAN-2024 Page. 24

You might also like